K
Khách

Hãy nhập câu hỏi của bạn vào đây, nếu là tài khoản VIP, bạn sẽ được ưu tiên trả lời.

Sửa đề:  Cho a, b, c là các số thực dương thỏa mãn điều kiện abc=1. Chứng minh rằng

\(\frac{1}{ab+b+2}+\frac{1}{bc+c+2}+\frac{1}{ca+a+2}\le\frac{3}{4}\)

Áp dụng bđt Cauchy-Schwarz ta có:

\(\frac{1}{ab+b+2}=\frac{1}{ab+1+b+1}\le\frac{1}{4}\left(\frac{1}{ab+1}+\frac{1}{b+1}\right)\) \(=\frac{1}{4}\left(\frac{abc}{ab\left(1+c\right)}+\frac{1}{b+1}\right)=\frac{1}{4}\left(\frac{c}{1+c}+\frac{1}{b+1}\right)\)

Tương tự \(\frac{1}{bc+c+2}\le\frac{1}{4}\left(\frac{a}{a+1}+\frac{1}{c+1}\right)\)

          \(\frac{1}{ca+a+2}\le\frac{1}{4}\left(\frac{b}{b+1}+\frac{1}{a+1}\right)\)

Cộng từng vế các bđt trên ta được

\(VT\le\frac{1}{4}\left(\frac{a+1}{a+1}+\frac{b+1}{b+1}+\frac{c+1}{c+1}\right)=\frac{3}{4}\)

Vậy bđt được chứng minh

Dấu "=" xảy ra khi a=b=c=1

4 tháng 8 2020

Vào thống kê hỏi đáp xem nhé. Bài này chỉ cần biểu diễn dưới dạng tổng bình phương là xong.

4 tháng 8 2020

ta có \(\frac{a^3}{b^2+3}+\frac{b^3}{c^2+3}+\frac{c^3}{a^2+3}\ge\frac{3}{4}\) (***)

do ab+bc+ca=3 nên

VT (***)=\(\frac{a^3}{b^2+ab+bc+ca}+\frac{b^3}{c^2+ab+bc+ca}+\frac{c^3}{a^2+ab+bc+ca}\)

\(=\frac{a^3}{\left(b+c\right)\left(a+b\right)}+\frac{b^3}{\left(c+a\right)\left(b+c\right)}+\frac{c^3}{\left(a+b\right)\left(c+a\right)}\)

áp dụng bđt AM-GM ta có \(\frac{a^3}{\left(b+c\right)\left(c+a\right)}+\frac{b+c}{8}+\frac{a+b}{8}\ge\frac{3a}{4}\)

\(\Rightarrow\frac{a^3}{\left(b+c\right)\left(c+a\right)}\ge\frac{5a-2b-c}{8}\left(1\right)\)

chứng minh tương tự ta cũng được

\(\hept{\begin{cases}\frac{b^3}{\left(c+a\right)\left(a+b\right)}\ge\frac{5b-2c-a}{8}\left(2\right)\\\frac{c^3}{\left(a+b\right)\left(c+a\right)}\ge\frac{5c-2a-b}{8}\left(3\right)\end{cases}}\)

cộng theo vế với vế của (1),(2) và (3) ta được VT (***) \(\ge\frac{a+b+c}{4}\)

mặt khác ta dễ dàng chứng minh được \(a+b+c\ge\sqrt{3\left(ab+bc+ca\right)}=3\)

đẳng thức xảy ra khi a=b=c=1 (đpcm)

24 tháng 4 2017

Nhân 2 vế của \(\frac{1}{a}+\frac{1}{b}+\frac{1}{c}=1\) có: \(ab+bc+ca=abc\)

Ta có: 

\(\frac{a^2}{a+bc}=\frac{a^3}{a^2+abc}=\frac{a^3}{a^2+ab+bc+ca}=\frac{a^3}{\left(a+b\right)\left(a+c\right)}\)

Áp dụng BĐT AM-GM ta có:

\(\frac{a^2}{a+bc}=\frac{a^3}{\left(a+b\right)\left(a+c\right)}+\frac{a+b}{8}+\frac{a+c}{8}\)

\(\ge3\sqrt[3]{\frac{a^3}{\left(a+b\right)\left(a+c\right)}\cdot\frac{a+b}{8}\cdot\frac{a+c}{8}}=\frac{3a}{4}\)

Tương tự cho 2 BĐT còn lại ta có:

\(\frac{b^2}{b+ca}+\frac{a+b}{8}+\frac{b+c}{8}\ge\frac{3b}{4};\frac{c^2}{c+ab}+\frac{a+c}{8}+\frac{b+c}{8}\ge\frac{3c}{4}\)

Cộng theo vế 3 BĐT trên ta có:

\(VT+\frac{4\left(a+b+c\right)}{8}\ge\frac{3\left(a+b+c\right)}{4}\)

\(\Leftrightarrow VT+\frac{4\left(a+b+c\right)}{8}\ge\frac{6\left(a+b+c\right)}{8}\)

\(\Leftrightarrow VT\ge\frac{a+b+c}{4}=VP\). Ta có ĐPCM

(

hhhhhhhhhhhhhhhhhhhhhhhhhhhhhhhhhhhhhhhhhhhhhhhhhhhhhhhhhhhhh

hhhhhhhhhhhhhhhhh

hhhhhhhhhhhhhhhhhh

hhhhhhhhhhhhhhh

hhhhhhhhhhhhh

23 tháng 1 2020

\(A=\frac{\frac{1}{2}a^2\left(\sqrt[3]{b}+\sqrt[3]{c}+1\right)\left[\left(\sqrt[3]{b}-\sqrt[3]{c}\right)^2+\left(\sqrt[3]{b}-1\right)^2+\left(\sqrt[3]{c}-1\right)^2\right]}{2\left(a+2\right)\left(a+\sqrt[3]{bc}\right)}\ge0\)

\(\Sigma_{cyc}\frac{a^2}{a+\sqrt[3]{bc}}=\Sigma_{cyc}A+\Sigma_{cyc}\frac{2\left(a-1\right)^2}{3\left(a+2\right)}+\frac{5}{6}\left(a+b+c\right)-1\ge\frac{5}{6}\left(a+b+c\right)-1=\frac{3}{2}\)

23 tháng 1 2020

Áp dụng bất đẳng thức cộng mẫu số 

\(\Rightarrow\frac{a^2}{a+\sqrt[3]{bc}}+\frac{b^2}{b+\sqrt[3]{ca}}+\frac{c^2}{c+\sqrt[3]{ab}}\)\(\ge\frac{\left(a+b+c\right)^2}{a+b+c+\sqrt[3]{bc}+\sqrt[3]{ca}+\sqrt[3]{ab}}\)

\(\Rightarrow\frac{a^2}{a+\sqrt[3]{bc}}+\frac{b^2}{b+\sqrt[3]{ca}}+\frac{c^2}{c+\sqrt[3]{ab}}\)\(\ge\frac{9}{3+\sqrt[3]{bc}+\sqrt[3]{ca}+\sqrt[3]{ab}}\)

Chứng minh rằng : \(\frac{9}{3+\sqrt[3]{bc}+\sqrt[3]{ca}+\sqrt[3]{ab}}\ge\frac{3}{2}\)

\(\Leftrightarrow18\ge3\left(3+\sqrt[3]{bc}+\sqrt[3]{ca}+\sqrt[3]{ab}\right)\)

\(\Leftrightarrow18\ge9+3\sqrt[3]{bc}+3\sqrt[3]{ca}+3\sqrt[3]{ab}\)

\(\Leftrightarrow9\ge3\sqrt[3]{ab}+3\sqrt[3]{bc}+3\sqrt[3]{ca}\)

Áp dụng bất đẳng thức Cauchy cho 3 bộ số thực không âm

\(\Rightarrow\hept{\begin{cases}a+b+1\ge3\sqrt[3]{ab}\\b+c+1\ge3\sqrt[3]{bc}\\c+a+1\ge3\sqrt[3]{ca}\end{cases}}\)

\(\Rightarrow2\left(a+b+c\right)+3\ge3\sqrt[3]{ab}+3\sqrt[3]{bc}+3\sqrt[3]{ca}\)

\(\Rightarrow9\ge3\sqrt[3]{ab}+3\sqrt[3]{bc}+3\sqrt[3]{ca}\left(đpcm\right)\)

Vì \(\frac{9}{3+\sqrt[3]{bc}+\sqrt[3]{ca}+\sqrt[3]{ab}}\ge\frac{3}{2}\)

Mà \(\frac{a^2}{a+\sqrt[3]{bc}}+\frac{b^2}{b+\sqrt[3]{ca}}+\frac{c^2}{c+\sqrt[3]{ab}}\ge\frac{9}{3+\sqrt[3]{bc}+\sqrt[3]{ca}+\sqrt[3]{ab}}\)

\(\Rightarrow\frac{a^2}{a+\sqrt[3]{bc}}+\frac{b^2}{b+\sqrt[3]{ca}}+\frac{c^2}{c+\sqrt[3]{ab}}\ge\frac{3}{2}\left(đpcm\right)\)

Chúc bạn học tốt !!!

21 tháng 7 2018

Ta có: \(a^2+b^2\ge2ab\)

\(\Rightarrow\frac{ab}{a^2+b^2}\le\frac{1}{2}\)

Tương tự cộng lại suy ra \(VT\le\frac{3}{2}\)

Suy ra sai đề :)

8 tháng 6 2020

Áp dụng BĐT AM-GM ta có: \(\frac{a}{b^3+ab}=\frac{1}{b}-\frac{b}{a+b^2}\ge\frac{1}{b}-\frac{b}{2\sqrt{ab^2}}=\frac{1}{b}-\frac{1}{2\sqrt{a}}\ge\frac{1}{b}-\frac{1}{4}\left(\frac{1}{a}+1\right)\)

Tương tự có: \(\hept{\begin{cases}\frac{b}{c^3+ca}\ge\frac{1}{c}-\frac{1}{4}\left(\frac{1}{b}+1\right)\\\frac{c}{a^3+ca}\ge\frac{1}{a}-\frac{1}{4}\left(\frac{1}{c}+1\right)\end{cases}}\)

Cộng 3 vế BĐT ta được:  \(\frac{a}{b^3+ab}+\frac{b}{c^3+bc}+\frac{c}{a^3+ca}\ge\frac{3}{4}\left(\frac{1}{a}+\frac{1}{b}+\frac{1}{c}\right)-\frac{3}{4}\)

Bài toán quy về chứng minh \(\frac{3}{4}\left(\frac{1}{a}+\frac{1}{b}+\frac{1}{c}\right)-\frac{3}{4}\ge\frac{3}{2}\)

\(\Leftrightarrow\frac{1}{a}+\frac{1}{b}+\frac{1}{c}\ge3\Leftrightarrow\left(\frac{1}{a}+a\right)\left(\frac{1}{b}+b\right)\left(\frac{1}{c}+c\right)\ge3+a+b+c=6\)

BĐT cuối hiển nhiên đúng vì theo BĐT AM-GM ta có:

\(\hept{\begin{cases}\frac{1}{a}+a\ge2\\\frac{1}{b}+b\ge2\\\frac{1}{c}+c\ge2\end{cases}}\)

Dấu "=" xảy ra <=> a=b=c=1

8 tháng 6 2020

\(\frac{a}{b^3+ab}+\frac{b}{c^3+bc}+\frac{c}{a^3+ac}\)

\(=\frac{a}{b\left(b^2+a\right)}+\frac{b}{c\left(c^2+b\right)}+\frac{c}{a\left(a^2+c\right)}\)

\(=\frac{1}{b}-\frac{b}{b^2+a}+\frac{1}{c}-\frac{c}{c^2+b}+\frac{1}{a}-\frac{a}{a^2+c}\)

\(\ge\frac{1}{b}-\frac{b}{2b\sqrt{a}}+\frac{1}{c}-\frac{c}{2c\sqrt{b}}+\frac{1}{a}-\frac{a}{2a\sqrt{c}}\)

\(=\frac{3}{4}\left(\frac{1}{a}+\frac{1}{b}+\frac{1}{c}\right)+\frac{1}{4}\left(\frac{1}{a}-\frac{2}{\sqrt{a}}+1\right)+\frac{1}{4}\left(\frac{1}{b}-\frac{2}{\sqrt{b}}+1\right)+\frac{1}{4}\left(\frac{1}{c}-\frac{1}{\sqrt{c}}+1\right)\)\(-\frac{3}{4}\)

\(\ge\frac{3}{4}.\frac{9}{a+b+c}+\frac{1}{4}\left(\frac{1}{\sqrt{a}}-1\right)^2+\frac{1}{4}\left(\frac{1}{\sqrt{b}}-1\right)^2+\frac{1}{4}\left(\frac{1}{\sqrt{b}}-1\right)^2-\frac{3}{4}\)

\(\ge\frac{3}{2}\)

Dấu "=" xảy ra <=> a = b = c = 1.

4 tháng 11 2019

CM BĐT : \(\left(x^2+y^2+z^2\right)^2\ge3\left(x^3y+y^3z+z^3x\right)\)   ( * )

\(\frac{a}{ab+1}=\frac{a\left(ab+1\right)-a^2b}{ab+1}=a-\frac{a^2b}{ab+1}\)

TT ....

Áp dụng BĐT ( * ) với x = \(\sqrt{a}\); y = \(\sqrt{b}\); z = \(\sqrt{c}\) vào bài toán, ta có :

\(\frac{a}{ab+1}+\frac{b}{bc+1}+\frac{c}{ca+1}=a+b+c-\frac{a^2b}{ab+1}-\frac{b^2c}{bc+1}-\frac{c^2a}{ac+1}\)

\(\ge3-\frac{a^2b}{2\sqrt{ab}}-\frac{b^2c}{2\sqrt{bc}}-\frac{c^2a}{2\sqrt{ac}}=3-\frac{\sqrt{a^3b}+\sqrt{b^3c}+\sqrt{c^3a}}{2}\ge3-\frac{\frac{\left(a+b+c\right)^2}{3}}{2}=\frac{3}{2}\)

Dấu " = " xảy ra \(\Leftrightarrow a=b=c=1\)